Calculus based kinematics (two dimensions) question

Click For Summary
A soccer ball is kicked at a 30° angle with a speed of 30 m/s, prompting calculations for flight duration, range, and maximum height. The velocity components are determined as 15√3 m/s for horizontal and 15 m/s for vertical motion. The maximum height is reached when the vertical velocity is zero, calculated as t = 15/9.8 seconds, while the total flight duration is t = 15/4.9 seconds. The range is found to be approximately 79.53 m, and the maximum height is confirmed at about 11.48 m. The discussion highlights the challenge of performing these calculations without a calculator, suggesting simplification methods to ease the process.
RJLiberator
Gold Member
Messages
1,094
Reaction score
63

Homework Statement


A soccer ball is kicked from the ground at an angle of 30◦ with speed 30m/s. Determine the
duration of the flight, range and maximal height of the trajectory.

Homework Equations

The Attempt at a Solution


1. Break down the velocity vector into x and y components:
30cos(30) = 15sqrt(3) m/s
30sin(30) = 15m/s

r(t) = (0,0)+t(15sqrt(3), 15)+(t^2)/2(0,-9.80)

This equation represents the position function.

If I take the derivative to get the velocity function, it becomes
r'(t) = (15sqrt(3), 15-9.8t)

When velocity in the y direction is 0, then the ball is at max height
so 15-9.8t=0
t=15/9.8 for max height.

When position of y = 0 the ball reaches its distance and final duration so:
15t-4.9t^2=0
t=0 or t= 15/4.9
clearly it is not t=0.

since we have time of duration:
t*velocity in x = range
15/4.9*15sqrt(3) = 79.5329 m

max height is found by dividing the time of max range by 2 so thew time of max height = 15/9.8 and then using that as t in the r(t) position function and finding the y-component.
The y-component comes out to be 11.48m. Seems reasonable.
15*15/98-9.8/2*(15/9.8)^2=11.48m

Am I doing this right? My problem is that our calculus class requires no calculators. To me, it seems like this problem is begging the use of calculators unless I just don't simplify things (which would be fine to ). It just seems like I am missing something that'd make the calculations easier.

Thanks.
 
Last edited:
Physics news on Phys.org
Everything looks right but you really calculated the max height time twice. Once by setting the velocity equation r'y = 0, you solved and got t = 15/9.8

The other by solving for the full time t = 15/4.9 then dividing by 2.

These are the same times so you only had to go wih one method. If you want simpler calculations, leave the 9.8 as g so you only have to multiply it through at the way end (or leave it in terms of g if your teacher doesn't mind)
 
  • Like
Likes RJLiberator
Damn, so since everything is correct, I have to assume they either want it simplified via use of a calculator (forbidden generally in this course) or unsimplified and a very whacky answer.

I guess it is good that I understand the problem :P.

I will have to talk to my instructor about this.
 
RJLiberator said:
15*15/98-9.8/2*(15/9.8)^2=11.48m

RJLiberator said:
Damn, so since everything is correct, I have to assume they either want it simplified via use of a calculator (forbidden generally in this course) or unsimplified and a very whacky answer.

I guess it is good that I understand the problem :P.

I will have to talk to my instructor about this.

You could go a bit further in terms of simplifcation w/o calculator:
Using y = vot + (1/2)at2
y = 15(\frac{15}{9.8}) - \frac{9.8}{2}(\frac{15}{9.8})^2

Cancel out the 9.8's on the right term, collect the squared terms:
y = (\frac{15^2}{9.8}) - \frac{1}{2}(\frac{15^2}{9.8})

Factor our (15^2)/9.8 which gives us:
y = \frac{225}{9.8}(1 - \frac{1}{2})
y = \frac{225}{19.6}

And then a little long division to finish it off? :D
 
  • Like
Likes RJLiberator
Question: A clock's minute hand has length 4 and its hour hand has length 3. What is the distance between the tips at the moment when it is increasing most rapidly?(Putnam Exam Question) Answer: Making assumption that both the hands moves at constant angular velocities, the answer is ## \sqrt{7} .## But don't you think this assumption is somewhat doubtful and wrong?

Similar threads

  • · Replies 2 ·
Replies
2
Views
2K
  • · Replies 3 ·
Replies
3
Views
1K
  • · Replies 38 ·
2
Replies
38
Views
4K
  • · Replies 2 ·
Replies
2
Views
1K
  • · Replies 4 ·
Replies
4
Views
2K
  • · Replies 7 ·
Replies
7
Views
2K
  • · Replies 5 ·
Replies
5
Views
3K
  • · Replies 4 ·
Replies
4
Views
1K
  • · Replies 10 ·
Replies
10
Views
2K
  • · Replies 10 ·
Replies
10
Views
3K